A client is admitted to the emergency department following a fall from a horse

1. Answer:B

SDHs are crescent in shape (see Figure 18.26) and are more common in patients with brain atrophy, including older adult and alcoholic patients, because the bridging vessels that cause these bleeds traverse greater distances in these patients. The bleed occurs between the dura and brain and is usually caused by acceleration-deceleration injuries. SDH is more common than epidural hematoma, occurring in up to 30% of patients with severe head trauma.

Figure 18.26

A client is admitted to the emergency department following a fall from a horse

Head computed tomography.

A biconvex-shaped density refers to an epidural hematoma. Most epidural hematomas result from a direct impact injury that causes a forceful deformity of the skull. An epidural hematoma is less likely in this patient because it is primarily a disease of the young and is rare in older adults and children younger than 2 years.

White densities in the cisterns and sulci suggest a traumatic subarachnoid hemorrhage. Subarachnoid hemorrhage is common in patients with severe head injury and is associated with early mortality. Severe head trauma is described as a GCS score of 8 or lower. This patient has a GCS score of 12, which makes this diagnosis less likely.

A normal head CT can be seen in patients with mild head injury, GCS score of 13 to 15, or severe head injury who have diffuse axonal injury that is often not apparent on initial injury. Given that this patient has moderate head injury based on his GCS score, these diagnoses are less likely.

Test-taking tip: When faced with a question that asks the most likely diagnosis, consider demographics (e.g., age, gender) and mechanism/situation to settle on the best answer.

2. Answer:C

Indications for emergent evacuation of an acute SDH include:

An acute SDH with a thickness greater than 10 mm or a midline shift greater than 5 mm on CT scan regardless of the patient’s GCS score

A comatose patient (GCS score <9) if the GCS score decreases between the time of injury and hospital admission by 2 or more points

A comatose patient who has asymmetric or fixed and dilated pupils on presentation

A comatose patient whose intracranial pressure exceeds 20 mm Hg

Answer C is the only one that falls into these criteria.

Choices A and B are both of patients with mild to moderate head injury based on GCS who do not have findings on CT that would require emergent surgical evacuation.

The presence of an acute SDH in a patient on antiplatelet therapy is not an indication for emergent surgical evacuation unless the previously listed conditions exist. Antiplatelet therapy is, however, associated with delayed acute SDH, which is defined as an acute SDH that is not apparent on initial CT scan but appears on a follow-up CT. Seventy percent of these patients experience neurologic deterioration within 24 hours.

Test-taking tip: Even if you do not know the guidelines, this question can be figured out using clinical gestalt. The patient describe in C is the only critically injured patient with a worsening GCS.

3. Answer:C

This is a difficult question because all of the answers are potential findings with a basilar skull fracture. Typical findings include raccoon eyes (bilateral periorbital ecchymosis), conjunctiva hemorrhage, anosmia, Battle’s signs (ecchymosis behind the ears), vision changes, cerebrospinal fluid rhinorrhea or otorrhea, step-off supraorbital edge, hearing loss, facial paralysis, and/or facial numbness.

Battle’s sign, however, is the only physical exam finding that is 100% associated with basilar skull fractures.

The other three findings (raccoon eyes, conjunctival hemorrhage, and vision changes) are less specific and can be associated with other facial injuries, including facial fractures and direct ocular trauma.

Test-taking tip: If you are unsure of the answer, look for the one that is most dissimilar to the others. This is not a perfect rule but can be helpful.

4. Answer:D

This patient has suffered a concussion, which is the mildest form of TBI. It is defined by a transient alteration of neurologic function caused by nonpenetrating injury to the brain and characterized by normal imaging studies, if they are obtained. These patients should not return to sports until they are symptom free and have been cleared by their primary care provider or a specialist in concussion management.

A CT scan is not indicated in the patient if the PECARN decision rule for head injury in trauma is used. For ages 2 to 18 years, if the patient has a GCS score less than or equal to 14 or signs of a basilar skull fracture, then a CT of the head is indicated. Also, if the patients has one or more of the following, CT is indicated: repeated vomiting after the injury, loss of consciousness, significant mechanism (e.g., pedestrian vs. auto, fall from greater than 5 feet), or severe headache.

Given that this child does not have an acute injury requiring admission or specialty consultation, involving pediatrics would not be helpful.

Test-taking tip: With choices C and D, you have two opposites; this suggests that one of them is probably the correct answer.

5. Answer:D

This patient has a septal hematoma. Although a relatively rare injury, it is an important one to recognize because it can lead to significant morbidity, including abscesses, septal perforation, or saddle nose deformity, if left untreated.

Given the potential morbidity of a septal hematoma, immediate treatment in the ED with incision and drainage is the correct answer rather than referral to an ENT that might lead to delay in care and complications.

In adults, these injuries typically occur with significant facial trauma and nasal fracture, which might warrant further imaging, including a maxillofacial CT. However, in children, nasal septal hematoma may be found with minor nasal trauma, such as simple falls, collisions with stationary objects, or minor altercations with siblings, because their septum is thicker and more flexible. With a minor mechanism, such as in this case, advanced imaging is not warranted.

Nasal packing is not indicated in septal hematoma but rather is used in epistaxis that cannot be managed by less invasive treatment (e.g., direct pressure, topical agents).

Test-taking tip: Try to eliminate the answers that are most likely incorrect such as nasal packing (not used in this setting) and obtaining a CT (not usually indicated in minor trauma). This leaves you with an answer that requires action in the ED versus one that results in delay of care. Action in the ED will usually win out as the correct answer.

6. Answer:A

Orbital blowout fractures are usually the result of a direct blow to the orbit. This causes a sudden increase in the intraorbital pressure, which in turn causes decompression by fracture of one or more of the bounding walls of the orbit.

The inferior orbital wall (see Figure 18.27) is the most commonly injured in an orbital blowout fracture because it is the thinnest wall, followed by the medial wall. Fractures of the lateral and superior walls are uncommon.

Figure 18.27

A client is admitted to the emergency department following a fall from a horse

Head computed tomography.

Associated clinical findings with orbital blowout fractures include enophthalmos due to increased orbital volume; diplopia due to extraocular muscle entrapment; orbital emphysema, especially when the fracture extends into an adjacent paranasal sinus; and/or malar region numbness due to injury to the inferior orbital nerve.

Test-taking tip: This is a “you know it or don’t” kind of question. If you don’t know it, choose an answer and move on. You can perhaps mark it for review when you are done at the end of the test. Don’t, however, let these kinds of questions get you stuck and waste your time.

7. Answer:A

The inferior orbital nerve is the most commonly injured nerve in an orbital blowout fracture. It courses within the bony floor of the orbit (the inferior wall), which is the wall most likely to fracture in this setting because of its relative thinnest compared with the other orbital walls.

The anterior-superior alveolar nerve runs in a canal in the anterior wall of the maxillary sinus, making injury to this nerve less likely in an isolated orbital blowout fracture.

The zygomatic branch of the facial nerve runs across the zygomatic bone to the lateral angle of the orbit, again making it less likely to be injured in a blowout fracture, where the inferior and medial walls are most likely to be injured.

The oculomotor nerve enters the orbit through the superior orbital fissure, again making it less likely to be injured with a blowout fracture.

Test-taking tip: Even if you do not know the anatomy and the correct answer to this question, you may be able to guess given the question is about an orbital blowout fracture and the only answer with “orbital” in it is the correct answer, A.

8. Answer:B

The clinical description of this patient presentation is consistent with a Le Fort II injury (Figure 18.28). A Le Fort II fracture is typically bilateral and pyramidal in shape. It extends superiorly in the midface to include fractures of the nasal bridge, maxilla, lacrimal bones, orbital floor, and rim. The nasal complex moves as a unit with the maxilla when the teeth are grasped and rocked.

Figure 18.28

A client is admitted to the emergency department following a fall from a horse

Le Fort fracture classification.

A Le Fort fracture I is a transverse fracture separating the body of the maxilla from the pterygoid plate and nasal septum. The hard palate and teeth would move but not the nose.

A Le Fort III fracture, which is rare, involves craniofacial dissociation when the entire face is separated from the skull due to fractures of the frontozygomatic suture line, across the orbit, and through the base of the nose and ethmoids. With this fracture, the entire face shifts, with the globes held in place only by the optic nerves.

A Le Fort IV fracture is similar to a Le Fort III but with the frontal bone involve.

Test-taking tip: This is a “you know it or you don’t” question. Questions regarding fracture classifications and types are common on emergency medicine standardized tests, so they are worth reviewing before the test.

9. Answer:B

This patient presents with a mandible fracture. Fractures of the mandible can result from any significant force, and because of the mandibles shape, multiple fractures may result from a single blow. This patient has overlying lacerations and mucosal tears suggesting an open fracture. Open mandible fractures require IV antibiotics that cover for oral flora, in this case, penicillin, and frequently require hospital admission. An oral maxillofacial surgeon should be consulted.

Choices A and B are not the most appropriate management of the patient because they do not involve IV antibiotics and consultation with a facial surgeon.

Although obtaining a facial CT scan may help further characterize the injury and help discover occult injuries, it is not the most appropriate management.

Test-taking tip: When the question asks for the most appropriate management, it should clue you to think of what is the standard of care for that particular condition.

10. Answer:C

This patient presents with classic signs and symptoms of tension pneumothorax. He is hypotensive, tachycardic, and tachypneic; he does not have breath sounds over the left hemothorax; and his trachea deviates away from the affected side. The most appropriate initial management of this patient is needle decompression of the left side of the chest followed shortly by chest tube placement.

Tension pneumothorax is an emergency and is a clinical diagnosis, so a chest radiograph should not be obtained before treatment. A chest tube on the affected side is indicated but only after needle decompression, to convert the tension pneumothorax to a simple pneumothorax, has been performed.

The patient on initial presentation is speaking and has an intact airway, so he does not require emergent intubation.

Test-taking tip: With questions about the initial management of trauma, think of the primary survey and identifying and treating life threats.

11. Answer:C

In the absence of associated injuries, most patients with isolated sternal fractures who can achieve adequate pain control on oral medications can safely be discharged home.

Isolated sternal fractures, especially those only diagnosed on CT, are not a marker for blunt cardiac injury; therefore, a stat echocardiogram would not be indicated in this patient, nor would admission for cardiac monitoring. Without associated injuries and with normal vital signs, this patient does not require admission to a surgical service.

Test-taking tip: If one answer is opposite to all of the others (one suggests discharge home and the others point to further testing/admission), the answer that stands out as different from the others is probably correct.

12. Answer:C

This patient presents with a significant mechanism and a mildly displace sternal fracture on lateral chest radiograph. This should raise the clinician’s level of concern for a cardiac contusion/blunt cardiac injury. Although there is no gold standard for the diagnosis of cardiac contusion, EKG is the most commonly recommended tool to screen for this injury. A normal EKG has an excellent negative predictive value (NPV) of greater than 95%. The addition of a negative troponin I to a normal EKG increases the NPV to almost 100%. Patient’s with a normal EKG and negative troponin I do not need any further cardiac workup or monitoring.

Although cardiac CT may help to differentiate traumatic from ischemic injury, it is not a good initial screening tool. An echocardiogram can be a very useful tool to rule out structurally significant myocardial injuries but should not be used as a primary screening modality for blunt cardiac injury. A troponin alone without an EKG is not a sensitive screening tool.

Test-taking tip: This is a stable patient in whom you are trying to rule out an injury, so think least invasive and least expensive.

13. Answer:C

This patient has suffered a unilateral facet dislocation but exhibits no neurologic deficits and has no other associated injuries. Unilateral facet dislocations are considered stable and do not require urgent neurosurgical evaluation. This patient can be discharged with referral to neurosurgery.

In the absence of neurologic deficits suggesting underlying spinal cord injury, an MRI is not warranted. This patient has no clinical finding concerning for vascular injury in the neck (potential arterial hemorrhage from the neck/nose/mouth, cervical bruit, expanding cervical hematoma, or focal neurologic deficits), so a CTA of the neck is not indicated.

Test-taking tip: If one answer is opposite to all of the others (one suggests discharge home and the others point to further testing/admission), the answer that stands out as different from the others is probably correct.

14. Answer:A

Scapula fractures are very uncommon, representing less than 1% of all fractures. They are usually caused by high-energy trauma and should prompt suspicion for other associated injuries, including other associated orthopedic injuries (ribs, proximal humerus, and clavicle) and pulmonary injuries (pneumothorax, hemothorax, and contusions). The other fractures are not always associated with high-energy mechanisms or associated injuries.

Test-taking tip: You should be able to eliminate a spinal transverse fracture as an answer because it is distant from the lungs. Clavicle fractures are common low-mechanism injuries, which also makes it easy to exclude this answer. If you don’t know the correct answer, this at least narrows the options down to two.

15. Answer:C

PCC is the most appropriate treatment at this time. This patient has an SDH on his CT. This is a life-threatening bleed, and his coagulopathy should be reversed as quickly as possible. In this case administering PCC is the best treatment. PCC is an inactivated concentrate of factors II, IX, and X, with variable amounts of factor VII.

Although PO vitamin K can help to reverse coagulopathy due to warfarin use, it does not reverse the coagulopathy quickly enough. In the setting of a life-threatening bleed, IV vitamin K can be given in conjunction with PCC.

Cryoprecipitate and platelets are not indicated in the treatment of life-threatening bleeds in patients on warfarin. Cryoprecipitate can be used in bleeding due to hemophilia A, and platelet replacement is indicated in patients with bleeding and significant thrombocytopenia.

Test-taking tip: This is a “you know it or don’t” kind of question. If you don’t know it, choose an answer and move on. You can perhaps mark it for review when you are done at the end of the test. Don’t, however, let these kinds of questions get you stuck and waste your time.

16. Answer:D

This patient presents with stab wounds to the upper back and chest and is at high risk for a pneumothorax. Initial chest radiograph does not demonstrate a pneumothorax, but pneumothorax in stab wounds may be delayed for 4 to 6 hours. There is a 12% reported incidence of initially asymptomatic stab wounds to the chest that require delayed tube thoracostomy for hemothorax or pneumothorax. Therefore, this patient should not be discharged home but should be observed and a repeat chest radiograph obtained after 4 to 6 hours.

Given a normal chest radiograph and a patient in not significant distress, a tube thoracostomy does not need to be performed at this time. A needle decompression, which is the treatment for a suspected tension pneumothorax (hypotension, respiratory distress, and decreased breath sounds on the affected side) is also not indicated in this clinical setting.

Test-taking tip: The correct answer is predicated on correctly interpreting the imaging study as normal. There is a tendency to want to find radiographic abnormalities on imaging studies in test-taking situations. A normal study can also be informative.

17. Answer:D

Indications for ED resuscitative thoracotomy remain controversial, but some recommendations include penetrating torso trauma patients with less than 15 minutes of CPR, blunt trauma patients with less than 10 minutes of CPR, and patients in profound refractory shock due to trauma. Only the patient described in D presents with one of these indications.

The patient described in A is more consistent with a pneumothorax or tension pneumothorax for which needle decompression and/or tube thoracostomy would be the appropriate treatment.

The patient described in B was initially in shock but responded to resuscitation efforts so does not require ED resuscitative thoracotomy.

The patient described in C is outside of the time limits for ED resuscitative thoracotomy.

Test-taking tip: ED resuscitative thoracotomy is a heroic procedure that is not indicated in patients who have stabilized and that has limited value in patients with no signs of life.

18. Answer:D

The NEXUS cervical spine criteria are a decision rule that was created to avoid unnecessary cervical spine imaging. The NEXUS criteria are listed in Box 18.1.

Box 18.1

NEXUS Cervical Spine Criteria

Absence of midline cervical tenderness

Normal level of alertness and consciousness

No evidence of intoxication

Absence of focal neurologic deficit

Absence of painful distracting injury

Any patient who meets all of the criteria is deemed low enough risk to not have to undergo imaging of the cervical spine. Of the patients described, only D meets all the NEXUS criteria.

Test-taking tip: Decision rules are becoming more important in emergency medicine, so being aware of the most used (e.g., NEXUS, PECARN) rules is important.

19. Answer:C

This patient has a sucking chest wound on the right side, also called an open pneumothorax. These wounds result in a unidirectional flow of air into the pleural space during inspiration but do not allow air to escape during expiration because of tissue apposition surrounding the wound. If left untreated, these wounds can lead to a life-threatening tension pneumothorax.

The most appropriate initial treatment for this patient is a three-sided occlusive dressing. This dressing creates a one-way valve that allows air and blood to escape the wound, preventing reentry of air that can create a tension pneumothorax. After the dressing is applied, a chest radiograph should be obtained. A chest tube will also eventually need to be performed to treat the underlying pneumothorax.

Endotracheal intubation is not indicated in this patient initially because of a normal mentation and intact airway.

Test-taking tip: The wording of “most appropriate immediate management” suggests a potentially lifesaving therapy of which a chest radiograph is not. This helps to eliminate at least one answer if you are unsure of the correct answer.

20. Answer:C

The preferred site for a chest tube is the fifth intercostal space mid-axillary line. This is approximately the level of the nipple. The tube should go over the sixth rib to avoid the neurovascular bundle.

The second intercostal space, mid-clavicular line is a common site for a needle thoracostomy to be performed.

On expiration, the diaphragm rises to the fifth rib. Performing a tube thoracostomy at the sixth intercostal space would risk entering the peritoneum and causing injury to intraabdominal organs.

Test-taking tip: As long as you know the difference between a tube and needle thoracostomy, choice A should be easy to eliminate. Any procedure done on the chest wall is always over the rib to protect the neurovascular bundle. This leaves only two answers to choose from if you are unsure of the correct answer.

21. Answer:A

Radiography of the chest can be a valuable tool to screen for traumatic aortic rupture. A widened mediastinum, as seen in Figure 18.29, is the most sensitive sign and is found in the majority of aortic ruptures. Widened mediastinum, however, is not a very specific finding, and up to half of patients with blunt aortic injury do not have a widened mediastinum; thus, patients in whom there is a high suspicion of aortic injury should undergo a chest CT.

Figure 18.29

A client is admitted to the emergency department following a fall from a horse

Chest radiograph.

The possibility of blunt aortic injury should be considered in all patients who sustain a severe deceleration injury, especially patients involved in MVCs moving in excess of 45 mph with evident of blunt force trauma to the chest.

First rib and scapular injuries are both suggestive of a high-energy mechanism and are associated with multiple other injuries, including pneumothorax, hemothorax, pulmonary contusions, cardiac injuries, and abdominal injuries. These fractures, however, are not sensitive for the diagnosis of blunt aortic injury.

A hemothorax on chest radiograph is not sensitive for an aortic injury; however, after tube thoracostomy placement, if there is a large initial rush of bright red arterial blood or significant ongoing losses, an aortic or other large vessel injury should be suspected, and the patient should be emergently transferred to surgery.

Test-taking tip: Hemothorax is a relatively common finding after blunt chest trauma, and aortic injury is rare, so you should be able to eliminate this answer as a sensitive finding.

22. Answer:D

This patient presents with a mechanism and physical exam concerning for retrobulbar hematoma, or postseptal hematoma. This can lead to orbital compartment syndrome. An IOP higher than 40 mm Hg in the setting of trauma is considered an emergency, and a lateral canthotomy should be performed without delay. This should be performed by the emergency clinician unless there is immediate availability of an ophthalmologist.

CT of the orbits may be required to further delineate the injury and direct further treatment but is not an immediate, vision-saving modality.

The use of IOP-lowering agents such as IV carbonic anhydrase inhibitors, topical ß-blockers, α-agonists, and in some cases, IV mannitol may be indicated in this clinical setting before ischemia and vision loss occur. Oral carbonic anhydrase inhibitors (acetazolamide) are not indicated in this clinical setting, nor are analgesia and cool compresses alone.

Test-taking tip: Most appropriate immediate management suggests a life-, limb-, or in this case vison-saving procedure. Lateral canthotomy is the only answer of the four that meets this criterion.

23. Answer:A

This patient has a traumatic hyphema. Complications of a traumatic hyphema include corneal staining and elevated IOP. In patients with a traumatic hyphema, nonsteroidal antiinflammatory drugs (NSAIDs) should be avoided be these are antiplatelet agents and can lead to worsening bleeding.

Morphine is a reasonable agent to use in such patients if they have severe pain related to their injuries.

Topical and oral agents to decrease IOP may be indicated in these patients in consultation with an ophthalmologist.

Patients with uncomplicated hyphema can be discharged home with recommendations for gentle ambulation and head of the bed elevation. Admission is recommended for patients with hyphema greater that 50%, sickle cell trait, uncontrolled IOPs, and anticoagulated patients.

Test-taking tip: Acetazolamide and topical steroids are agents used to treat elevated IOPs, which is a complication of this pathology, so they can be eliminated as agents not to give.

24. Answer:D

This patient has suffered severe facial trauma as evidenced by the Le Fort III fracture. In addition to obtaining the appropriate consultation, this patient should also undergo blunt cerebrovascular injury screening (BCVI).

Both the Western and Eastern Trauma associations recommend patients with Le Fort II or III fractures undergo BCVI screening. A CTA of the neck is rapidly obtainable in most trauma centers and is the first-line test. However, a CTA can miss injuries, and in patients in whom there is a high degree of suspicion for injury, conventional arteriography should be obtained.

See Figure 18.28 for more detailed demonstration of the Le Fort classification.

CT of chest, abdomen, and pelvis is not indicated in this patient, who was only assaulted in the face and head and has no complaints or visible injury to the rest of his body.

Lethal complications have been reported after the insertion of a nasogastric tube after severe basilar skull fracture. It has thus been recommended that in the setting of basilar skull fracture and/or significant maxillofacial trauma, insertion of tubes in the nasopharynx should be avoided.

A soft tissue neck radiograph can be useful in a variety of clinical settings, including suspected epiglottitis, croup, retropharyngeal abscess, or airway foreign body. It does not have a role, however, in the setting of blunt head and neck trauma.

Test-taking tip: Choice A should be easy to eliminate given the clinical scenario, and most physicians have the picture of a nasogastric tube coiled in the brain burned into their memories, making choice B easy to discard.

25. Answer:D

The hangman’s fracture, which involves traumatic spondylolysis of C2, occurs when the skull, atlas, and axis, functioning as a unit, are hyperextended as the result of an abrupt deceleration. As the name suggest, it was originally described in victims of hanging injury but in modern society is more commonly associated with head-on MVC.

All of the other fractures listed are considered stable.

A unilateral facet dislocation occurs when simultaneous flexion and rotation occur, causing the contralateral facet joint to dislocate. The superior facet rides forward and over the tip of the inferior facet, coming to rest within the intervertebral foramen. Because the facet is locked, this is a stable injury.

A clay shoveler’s fracture, named for the fracture caused by the abrupt head flexion that clay miners might experience while lifting a heavy shovelful of clay, is an avulsion fracture of the spinous process. In modern society, this injury is seen with direct trauma to the neck or after sudden deceleration in an MVC that causes forced flexion of the neck. Because the injury only involves the spinous process, it is stable.

An isolated transverse process, similar to the clay shoveler’s fracture, is stable.

Test-taking tip: If you have to guess, the name “hangman’s fracture” might clue you to it being unstable because hanging can be a lethal mechanism.

26. Answer:A

This patient presents with a severe TBI after a fall. In patients with severe TBI, high-dose methylprednisolone is associated with increased mortality and is therefore contraindicated.

Mannitol can be administered at a rate of 0.25 to 1 mg/kg in patients with signs of progressive neurologic deterioration but is not associated with improved mortality or neurologic outcomes.

Levetiracetam can be used to prevent early post-traumatic seizures but is not associated with improved mortality.

Ketamine was classically thought to raise intracranial pressure, but that has not been found to be the case, and it can safely be used in patients with TBI.

Test-taking tip: Mannitol and levetiracetam are commonly used in head-injured patients so can easily be eliminated as the correct answer for this question.

27. Answer:C

SVT is the most common arrhythmia seen in TBI patients. Cardiac dysrhythmias in this setting are most often caused by high levels of circulating catecholamines. Ensuring adequate tissue perfusion is the primary goal in these patients, along with treating elevated intracranial pressure. The dysrhythmia will often resolve after these two issues are address.

A prolonged QT interval can be hereditary or can be acquired because of electrolyte abnormalities or the use of many common medications, including antibiotics, antihistamines, and antipsychotics. It is not associated with TBI.

First-degree atrioventricular block is defined as a prolongation of the PR interval beyond the upper limit of normal (0.02 second). It is often an incidental finding on EKG in asymptomatic patients and is usually benign. It is common in well-trained athletes but can also be caused by use of some medications, including calcium channel blockers and ß-blockers. It is not associated with TBI.

Ventricular tachycardia is the most common cause of sudden cardiac death in the United States and has many causes, the most common being cardiac ischemia. It can be seen in severe TBI but is not as common as SVT.

Test-taking tip: This is a “you know it or don’t” kind of question. If you don’t know it, choose an answer and move on. You can perhaps mark it for review when you are done at the end of the test. Don’t, however, let these kinds of questions get you stuck and waste your time.

28. Answer:C

This patient presents with a mild head injury, defined as a GCS score of 13 to 15, after a fall. In any pediatric patient with blunt head trauma, the PECARN pediatric head injury rule should be applied. In patients younger than 2 years the following are high-risk criteria: GCS score <14, altered mental status (agitation, somnolence, repetitive questioning, or slow response to verbal communication), and palpable skull fracture. If any of those are present, then a noncontrast brain CT is indicated.

The following are considered low-risk criteria: nonfrontal scalp hematoma, loss of consciousness for less than 5 seconds, severe injury mechanism, and abnormal activity per parents. If any of these criteria are met, then consider a period of observation versus head CT.

This patient does not have any of these criteria, and therefore CT of the head is not indicated. A period of observation in the ED might be warranted, but admission is not required.

Seizure prophylaxis is not indicated in mild head injury but may be indicated in severe head injury.

Test-taking tip: Brush up on the clinical decision rules, such as PECARN, before the exam.

29. Answer:B

This patient has motor function associated with C5 (shrugging of shoulders), but nothing caudal to this, suggesting a cord injury at C6 (see Table 18.1).

Table 18.1

Level of Spinal Injury and Associated Loss of Motor Function

LevelResulting Loss of Motor Function

C4

 

Spontaneous breathing

 

C5

 

Shrugging of shoulders

 

C6

 

Flexion at elbow

 

C7

 

Extension at elbow

 

C8–T1

 

Flexion of fingers

 

T1–12

 

Intercostal and abdominal muscles

 

L1–2

 

Flexion at hip

 

L3

 

Adduction at hip

 

L4

 

Abduction at hip

 

L5

 

Dorsiflexion of foot

 

S1–2

 

Plantar flexion of foot

 

S2–4

 

Rectal sphincter tone

 

LevelResulting Loss of Motor Function

C4

 

Spontaneous breathing

 

C5

 

Shrugging of shoulders

 

C6

 

Flexion at elbow

 

C7

 

Extension at elbow

 

C8–T1

 

Flexion of fingers

 

T1–12

 

Intercostal and abdominal muscles

 

L1–2

 

Flexion at hip

 

L3

 

Adduction at hip

 

L4

 

Abduction at hip

 

L5

 

Dorsiflexion of foot

 

S1–2

 

Plantar flexion of foot

 

S2–4

 

Rectal sphincter tone

 

Test-taking tip: Although memorizing all of the myotomes or dermatomes is probably not worth your time (unless you have a photographic memory), remember key ones (C4—breathing, C6—flexion at elbow, L1–2—flexion at hip, L5—dorsiflexion at hip) should get you close enough to get the right answer.

30. Answer:B

Aortic disruptions are life-threatening injuries that should always be suspected in any decelerating injury with significant energy. High-speed MVCs are the most common mechanism.

The most common site for aortic disruption or tear is the descending aorta at the isthmus just distal to the origin of the left subclavian artery. About 80% to 90% of aortic tears occur at this site.

Injuries at the other sites listed are less common but can occur. While injuries to the ascending aorta are less common, they are more likely to be lethal. Any patient in whom aortic injury is suspected should get a CT of the aorta if clinically stable and immediate cardiothoracic surgery consult.

Test-taking tip: This is a “you know it or don’t” kind of question. If you don’t know it, choose an answer and move on. You can perhaps mark it for review when you are done at the end of the test. Don’t, however, let these kinds of questions get you stuck and waste your time.

31. Answer:C

This patient has a pulmonary contusion. A pulmonary contusion is a direct injury to the lung as a result of blunt trauma. Patients with pulmonary contusions can present with chest pain, tachypnea, contusion to the chest wall, hypoxia, and shortness of breath.

Chest radiograph classically shows ground-glass opacities, although this can be delayed. Areas of opacification on chest imaging within 6 hours of blunt trauma can be considered diagnostic of pulmonary contusion. CT is more sensitive for pulmonary contusions than chest radiograph and has shown these to be common in blunt trauma and a source of severe morbidity and mortality.

Aggressive fluid resuscitation should be avoided because this can lead to worsening lung injury. Neither steroids nor antibiotics are indicated in pulmonary contusion.

Aggressive respiratory therapy, which can include positive pressure ventilation, is the most appropriate treatment for this patient.

Test-taking tip: The patient is normotensive, so aggressive fluid resuscitation should be easy to eliminate as an answer. Steroids are never indicated in the initial management of trauma, leaving two possible answers to choose from.

32. Answer:D

This patient presents with minor chest trauma after an MVC. The NEXUS chest CT decision instrument is designed to identify patients who suffered blunt chest trauma but do not need advanced imaging. It is highly sensitive for clinically identifying the risk for major thoracic injuries and therefore for excluding patients who do not need advanced imaging.

The criteria are for advanced imaging are abnormal chest radiograph, rapid deceleration mechanism (fall >20 feet, MVC >40 mph), distracting painful injury, chest wall tenderness, sternal tenderness, thoracic spine tenderness, and scapular tenderness. If all of these are negative, then the rate of major thoracic injury is <1% and CT is not indicated. This patient has none of these findings, and thus CT is not warranted.

This patient has a GCS score of 15 with no loss of consciousness, so a head CT is not warranted, and given no other identifiable injuries, a consultation with a trauma surgeon is also not necessary.

Test-taking tip: Don’t get fooled in a test-taking situation into believing something else has to be done. If the case scenario is low risk, sometimes less is better.

33. Answer:D

Immediate drainage of more than 1,500 mL of blood is a possible indication for urgent thoracotomy because it is suggestive of significant vascular injury. This is especially true in a patient in hemorrhage shock. Another potential indication is 200 mL drainage per hour for 3 hours.

The color of the blood is also important: dark, venous blood is more likely to cease spontaneously compared with bright arterial blood.

Test-taking tip: Operative management for hemothorax is relatively rare, so guessing the largest number is a safe bet.

34. Answer:C

IOP measurement should be avoided in this patient because he likely has an open globe injury as evidenced by the irregularly shaped pupil and severely decreased vision in the affected eye. Increased pressure on the globe that may be created by attempting to measure pressures may worsen the injury. Broad-spectrum antibiotics should be administered to patients with an open globe injury.

Although there was initially concern that ketamine increases IOP, which would be contraindicated in this patient, more recent evidence suggests that clinically this is not the case, and if this patient needed to be intubated or sedated it could be used.

A fluorescein exam is safe in this patient and, if performed, may show Seidel’s sign, which is considered diagnostic of an open globe injury.

Test-taking tip: The picture is of an open globe injury, which should raise the concern for avoiding increased IOP. Antibiotics and fluorescein do not do this so can easily be eliminated as the correct answer.

35. Answer:C

In hemodynamically unstable patients with blunt abdominal trauma, bedside ultrasound (when available) is the initial diagnostic modality of choice to identify the need for emergent laparotomy.

While CT of the abdomen and pelvis with IV contrast is the noninvasive gold standard for diagnosing abdominal injuries, it is not indicated or appropriate in unstable patients.

KUB is plain film radiographic imaging. While this modality may reveal some findings (such as subdiaphragmatic free air, bony injuries, or radiopaque foreign bodies), it is not the initial preferred imaging modality in the emergent setting.

DPL is 98% sensitive in determining intraabdominal bleeding or bowel injury that may require immediate laparotomy and should be considered in an unstable patient if FAST exam is not available, difficult to perform, or inconclusive. DPL is rapid to perform but is invasive and can miss diaphragmatic, retroperitoneal, or isolated hollow viscus injuries.

Test-taking tip: “Hypotensive” and “initial” are the key words in this question, indicating a fast bedside test that equals a bedside ultrasound/FAST exam.

36. Answer:C

Oral contrast is not essential to the evaluation of blunt abdominal trauma. While oral contrast has a theoretical advantage of improved identification of bowel injuries, pancreas injuries, and hematomas, it also has disadvantages. Disadvantages include vomiting, aspiration, and delayed diagnosis. Even with theoretical advantages, the sensitivity of CT does not differ significantly with or without oral contrast, and oral contrast is therefore not essential to the evaluation of blunt abdominal trauma.

Hemodynamically unstable patients with a positive FAST exam should go directly to the operating room for laparotomy.

IV contrast CT of the abdomen and pelvis (not IV, PO, and rectal contrast CT of the abdomen and pelvis) is the gold standard for evaluation of injuries in blunt abdominal trauma and has excellent sensitivity and specificity for intraabdominal injuries, particularly liver and spleen injuries. CT is less reliable in diagnosing bowel, mesenteric, pancreas, and diaphragmatic injuries.

DPL has a sensitivity of 98% for detecting intraabdominal injuries; however, DPL is not reliable in detecting bowel, diaphragmatic, and retroperitoneal injuries.

Test-taking tip: If you are unsure of the answer, you can eliminate choice A because unstable patients never belong in the CT scanner and choice D because DPL is rarely used anymore, leaving two answers to choose from.

37. Answer:C

Diaphragmatic injury is rare (0%–5% of patients with thoracoabdominal injury), may result from blunt or penetrating injuries, and is almost exclusively left-sided. Signs and symptoms are nonspecific, and delayed diagnosis may lead to herniation or to strangulation of abdominal contents.

Hiatal hernia refers to herniation of abdominal contents through the esophageal hiatus of the diaphragm and may be related to congenital malformations, surgical procedures, or in some cases, trauma.

Blebs (also known as bullae) are pulmonary lunacies often seen in lung disease. Ruptured blebs result in pneumothorax, not diaphragmatic injuries.

Pneumomediastinum (also known as mediastinal emphysema) is the presence of air or other gas in the mediastinum. Pneumomediastinum is categorized as spontaneous or traumatic. Traumatic pneumomediastinum can be the result of blunt or penetrating trauma, barotrauma from mechanical ventilation, or iatrogenic injury.

Test-taking tip: If you are unsure of the answer, you can eliminate hiatal hernia and ruptured bleb because they are not usually associated with trauma, leaving two answers to choose from.

38. Answer:A

Blunt injury to intestines can result from deceleration injuries and tearing of structures near a fixed point of attachment. Presence of a seat belt sign (linear ecchymosis of the abdominal wall) or presence of a lumbar distraction fracture (also known as a Chance fracture or transverse fracture through the vertebral body caused by flexion about an axis anterior to the vertebral column) may be associated with retroperitoneal and abdominal visceral injuries. Diagnosis may be difficult because injured structures may only have minimal bleeding, and early ultrasound and CT are often nondiagnostic. Observation and serial abdominal exams are indicated.

While orthopedics consult and treatment with thoracolumbosacral orthosis may ultimately be indicated for the Chance fracture, your more immediate next step should be evaluation of potential retroperitoneal and/or visceral injuries.

Evaluating for injuries with a DPL can identify intraabdominal bleeding/bowel injury that requires immediate laparotomy, is rapidly performed an d readily available, and has 98% sensitivity. DPL can miss significant retroperitoneal bleeding and isolated hollow viscus perforation and is contraindicated if there is indication for emergent laparotomy.

Emergent MRI is not indicated.

Test-taking tip: The key phrase in this questions is “most appropriate next step.” You have identified an injury that needs management, which should allow you to narrow down the correct answer to A or B. Choice A is most appropriate because it is more important to manage possible life threats, such as occult intraabdominal injuries.

39. Answer:B

Your patient has blunt abdominal trauma and is hemodynamically unstable, an indication for immediate surgical consultation and laparotomy. The spleen and then the liver are the most commonly injured intraabdominal organs following blunt trauma. Your patient’s left shoulder pain, known as Kehr’s sign, is a classic symptom of splenic injury and is a result of diaphragmatic irritation from hemorrhage.

CT of the abdomen and pelvis with IV contrast is the noninvasive gold standard for the diagnosis of abdominal injury, but your patient is hemodynamically unstable and should go directly to the operating room for laparotomy and definitive treatments.

In a patient with blunt trauma and unstable vital signs, death may occur as a result of massive hemorrhage. It is inappropriate to wait for blood transfusion before definitive treatment.

Your patient is unstable and needs immediate laparotomy. It is not appropriate to delay care.

Test-taking tip: Unstable trauma patients need a surgeon, making B the clear choice.

40. Answer:C

With penetrating stab wounds, the liver is the most commonly injured organ, followed by the small bowel.

In gunshot wounds, the small bowel is the most commonly injured organ, followed by the colon and then the liver.

Test-taking tip: This is a “you know it or don’t” kind of question. If you don’t know it, choose an answer and move on. You can perhaps mark it for review when you are done at the end of the test. Don’t, however, let these kinds of questions get you stuck and waste your time.

41. Answer:A

External tocodynamometric monitoring for a minimum of 4 to 6 hours of a potentially viable fetus is indicated for all pregnant patients evaluated for trauma, even those without obvious abdominal injury. If the patient has fewer than three contractions per hour during the initial 4-hour monitoring, she may be discharged home. If there are persistent contractions or uterine irritability, fetal monitoring may be extended to 24 hours. If there is evidence of fetal distress or uterine irritability during the initial assessment, then immediate obstetrician consultation should be obtained.

Serial abdominal exams may be indicated in patients after suffering abdominal trauma but are not enough in patients who have a potential viable fetus.

Pelvic examination after an ultrasound is obtained is indicated to determine placental location and to exclude placenta previa but would not be the next step or necessarily emergent. A sterile pelvic exam can identify injuries to the lower genital tract, vaginal bleeding, and rupture of amniotic membranes. Vaginal fluid with a pH of 7 is suggestive of amniotic fluid, and branch-like pattern (ferning) is suggestive of amniotic fluid.

A pregnant trauma patient with a potentially viable fetus, and even without obvious abdominal injury, should have 4 to 6 hours of external tocodynamometric monitoring and should not be discharged home without this monitoring even if they have appropriate follow-up and good return precautions.

Test-taking tip: This is a “you know it or don’t” kind of question. If you don’t know it, choose an answer and move on. You can perhaps mark it for review when you are done at the end of the test. Don’t, however, let these kinds of questions get you stuck and waste your time.

42. Answer:D

While imaging studies should be minimized to avoid fetal exposure to ionizing radiation, never withhold imaging needed for appropriate maternal trauma management. Do not withhold critical maternal interventions or diagnostic procedures out of concern for potential adverse fetal consequences. Fetal exposure can be decreased by shielding the maternal abdomen and pelvis, and modified studies and dose-reducing techniques may be obtained.

Tetanus toxoid is a category C drug in all trimesters of pregnancy, but it is commonly accepted as safe and should be provided as needed. Tetanus antibiotics cross the placenta and can reduce the incidence of neonatal tetanus.

Fetomaternal hemorrhage is the entry of fetal RBCs into the maternal blood stream, and as little as 0.1 µL of fetal Rh-positive blood entering an Rh-negative mother’s circulation can sensitize the mother and endanger the pregnancy (and future pregnancies). Rho(D) immunoglobulin should be administered to Rh-negative pregnant women with any abdominal trauma. There are two dose options: 50 mcg IM for gestation £12 weeks and 300 mcg IM for gestation of ≥13 weeks.

A gravid uterus may compress the maternal inferior vena cava in the supine position and may put the mother at risk for “supine hypotension syndrome,” with diminished venous return and decreased cardiac output. In an effort to minimize vena cava compression, all pregnant patients should be kept in the semi–left lateral decubitus position.

Test-taking tip: Words such as “always,” “never,” and “only” should clue you that this is probably not the correct answer. Absolute statements, such as these, are rarely true in life and in medicine.

43. Answer:A

Trauma is the leading cause of nonobstetric morbidity and mortality in pregnant women, with MVCs accounting for most blunt abdominal trauma, followed by falls and assault. Placental abruption can be seen in up to 5% of minor injuries and up to 50% of major traumatic injuries and is the second most common cause of fetal death (maternal death is the number one cause of fetal death).

During trauma, the elastic uterus deforms, and the inelastic placenta can shear from the uterine wall. The most sensitive clinical finding for placental abruption after trauma is uterine irritability (more than three contractions per hour). Other findings include abdominal pain, painful vaginal bleeding, and tetanic uterine contractions. Maternal complications of placenta abruption include disseminated intravascular coagulation and amniotic fluid embolism with the introduction of placental products into the maternal circulation.

Uterine rupture is rare (<1% of all injuries in pregnancy) and is more likely to be seen during the late second trimester and third trimester with direct impact on the uterus. Clinical findings are nonspecific and can include loss of palpable uterine contour, easily palpable fetal parts, or abnormal fetal location on radiology studies. Fetal mortality rate is high.

Fetal tachycardia and vaginal bleeding are nonspecific and can be found with numerous injuries.

Test-taking tip: Fetal tachycardia and vaginal bleeding, as stated previously, are very nonspecific findings in the setting of trauma in pregnancy; they thus should be easily eliminated as an answer looking for the most sensitive clinical finding.

44. Answer:D

Perimortem cesarean delivery is a rare procedure because cardiac arrest is reported to occur in only 1 in 30,000 pregnancies. Indication for perimortem cesarean delivery is loss of vital signs in a pregnancy that is at (or near) the age of viability. Perimortem cesarean delivery is ideally performed at the 4-minute mark following onset of maternal cardiac arrest with delivery of the infant at 5 minutes. There should be no delay in performing the potentially life-saving maneuver because survival of the infant is directly related to the time elapsed from maternal cardiac arrest to delivery. CPR should continue during and after perimortem cesarean delivery to monitor the effects of cesarean delivery and resultant relief of physiologic aortocaval obstruction and to monitor for return of spontaneous maternal circulation.

While it is appropriate to ensure that you have adequate vascular access in a patient who is coding, in a pregnant patient, you should not delay delivery of a potentially viable infant. Infant survival is best when perimortem cesarean delivery is performed within 5 minutes.

A bedside FAST exam and ultrasound for fetal evaluation is not indicated in a case of maternal cardiac arrest and would delay attempts at a perimortem cesarean delivery.

It is appropriate to consider fluid resuscitation and blood transfusion in trauma patients who present with hemodynamic instability; however, this pregnant patient is actively coding, and you should not delay delivery of a potentially viable infant.

Test-taking tip: This is an extreme situation that calls for extreme measures of which the answer to perform a perimortem cesarean delivery fits best.

45. Answer:C

Pelvic fractures in older adults, particularly women, may occur from low-energy falls from standing or seated positions. CT may only be 77% sensitive for pelvic fractures (particularly posterior), and MRI should be considered for patients with pelvic pain and pain with weight bearing, even with negative CT imaging.

It is not appropriate to discharge a patient home with uncontrolled pain and inability to bear weight. Your patient needs additional imaging (MRI) for further evaluation.

While admission for pain control is reasonable, the most appropriate next step in management is a pelvic MRI for further evaluation of continued pain and inability to bear weight.

Again, discharge is not appropriate in a patient with uncontrolled pain and inability to bear weight. Despite negative radiographs and negative CT imaging, an MRI is indicated for further evaluation of pelvic fracture in this older adult, osteopenic female.

Test-taking tip: The picture of sending an older adult woman home on crutches should make it easy to exclude this answer, and sending a patient to a skilled nursing facility without a diagnosis should also raise the hair on the back of your next, leaving only two possible answers to choose from.

46. Answer:C

This patient is hemodynamically unstable with a pelvic fracture. Other sources of bleeding have been excluded with a negative FAST exam and negative CT scan. She should undergo angiography with embolization. Most bleeding from pelvic fractures is due to low-pressure venous bleeding; however, shock and death are generally secondary to arterial bleeding. The arteries most commonly involved are branches of the internal iliac system, with the superior gluteal and obturator artery being the most common. Up to 4 L of blood can be accommodated in the pelvis until vascular pressure is overcome by tamponade.

CT is the gold standard for evaluating pelvic injuries and is more sensitive than plain radiographs.

In pelvic fractures, hemorrhage is the primary cause of mortality. Your patient is hemodynamically unstable with a documented pelvic fracture. Your primary goal should be hemorrhage control with angiography.

Delayed death is often caused by sepsis from open fractures. It is reasonable to carefully inspect the skin over the posterior pelvis, gluteal area, perineum, vagina, and rectum to evaluate for open injuries; however, in this case, it is more important to control hemorrhage then to perform a pelvic exam.

Test-taking tip: The patient scenario is one of a patient in hemorrhagic shock. The next appropriate step is one to stop the bleeding, making it easy to eliminate B and D as correct answers.

47. Answer:B

Bladder injury occurs in about 2% of blunt abdominal trauma patients, with 70% to 97% associated with pelvic fractures. Direct blows to a distended bladder (bladder distention is often seen in alcohol-intoxicated patients), with high-energy mechanism, should raise your index of suspicion for bladder rupture.

Abdominal swelling from urine ascites and perineal or scrotal edema from urinary extravasation are common.

Bladder ruptures are classified as intraperitoneal (occurring at the superior dome of the bladder) and extraperitoneal (occurring at the inferior aspect of the bladder).

A cystogram is the gold standard imaging study in the diagnosis of bladder rupture.

Urethral injuries can be seen in 5% to 10% of pelvic fractures and may be associated with blood at the meatus but are not usually associated with abdominal swelling and scrotal edema

The ureter is the least frequently injured GU organ. Isolated ureteral injury is rare in trauma patients because the ureter is well protected in the retroperitoneum. The majority of ureteral injuries are caused by penetrating trauma. There are no specific history or physical exam findings for ureteral injuries, and they can easily be missed.

The kidneys are protected by adjacent structures but are suspended from the renal pedicle, and blunt trauma or rapid deceleration injuries may result in injuries including contusions, shattering of the kidney, and vascular injuries that may lead to necrosis.

Test-taking tip: The lower genitourinary findings, including blood at the meatus and scrotal swelling, should help you eliminate C and D as answers.

48. Answer:D

In blunt abdominal trauma with stable vital signs and gross hematuria, defined as >50 RBCs/hpf, an IV contrast CT of the abdomen and pelvis is indicated for diagnosis of renal or GU injury.

In the setting of blunt abdominal trauma and gross hematuria, you should have a high index of suspicion for renal or GU injuries. Your patient needs further evaluation with an IV contrast CT of the abdomen and pelvis. It is not appropriate to “do nothing.”

Outpatient workup is not appropriate in this patient with blunt trauma and gross hematuria, although it may be appropriate in stable trauma patients with microscopic hematuria.

IV contrast CT of the abdomen and pelvis is the gold standard for assessing renal and GU trauma and is more sensitive and specific than an IVP.

Test-taking tip: The key to this question is realizing that a urinalysis with 75 RBCs/hpf represents gross hematuria. If you know this, then it is clear that the patient needs an immediate diagnostic test, which eliminates choices A and B.

49. Answer:A

Your patient hit her face on the bottom of the swimming pool, resulting in hyperextension of her neck. Her clinical findings are consistent with central cord syndrome. Central cord syndrome is the most common incomplete spinal cord lesion, caused by hyperextension. It is often seen in patients with preexisting osteoarthritic changes or cervical canal stenosis, making it more common in older patients. This hyperextension results in damage to the anterior spinal artery, which supplies the central portion of the spinal cord where motor fibers reside, resulting in weakness and numbness that are disproportionately greater in the arms than the legs.

Anterior cord syndrome is caused by flexion or extension with vascular injury of the anterior spinal artery or bony fragment injury and results in paralysis and loss of pain and temperature sensation. Position, touch, and vibration are preserved.

Brown-Sequard syndrome is the result of hemisection of the spinal cord, usually associated with penetrating trauma. This injury results in ipsilateral paralysis, loss of proprioception, and vibratory sensation. Contralateral loss of pain and temperature sensation is also seen.

Transient spinal shock refers to loss of muscle tone and reflexes seen after spinal cord injury. Duration of spinal shock is variable.

Test-taking tip: Brush up on your spinal cord syndromes before the exam because they are frequently tested.

50. Answer:C

The neck is defined by triangles, zones, and fascial planes. The anterior triangle (formed by the borders of the sternocleidomastoid, inferior mandible, and midline neck) contains the most vital structures and is further divided into the following horizontal zones:

Zone I: clavicles to the cricoid cartilage

Zone II: cricoid cartilage to the angle of the mandible

Zone III: angle of the mandible and base of the skull

Test-taking tip: Brush up on some basic anatomy before the test, especially as it is relevant to patient care.

51. Answer:B

In penetrating neck trauma, careful physical exam is more than 95% sensitive for detecting clinically significant vascular and aerodigestive injuries. Laryngotracheal injuries may present with stridor, dysphonia, air or bubbling in the wound, airway obstruction, hoarseness, subcutaneous emphysema, tracheal deviation, laryngeal edema/hematoma, and/or restricted vocal cord mobility.

Vascular injuries present with active bleeding, pulsatile or expanding hematomas, thrill or bruits, hypotension, or shock unresponsive to initial fluid therapy.

Pharyngoesophageal injuries present with odynophagia, dysphagia, hematemesis, blood in the mouth, saliva draining from the wound, and prevertebral air.

Test-taking tip: You should be able to easily eliminate A and C if you think about the anatomy of the neck.

52. Answer:B

Penetrating neck trauma may result in vascular injury and/or aerodigestive injuries. In a patient with penetrating neck trauma who is having painful swallowing, difficulty swallowing, blood in the mouth, saliva draining from the wound, or a trans-midline trajectory, you should have a high index of suspicion for pharyngoesophageal injury.

Esophagram or esophagoscopy should be expeditiously obtained because morbidity increases if repair is delayed by more than 24 hours. Combined esophagram and esophagoscopy have 100% sensitivity for pharyngoesophageal injury. Missed esophageal injuries have a high morbidity and mortality (up to 50%).

CTA should be considered for evaluation of vascular neck injuries, not pharyngoesophageal injuries.

Bronchoscopy is more appropriate for tracheobronchial injuries.

Color-flow Doppler ultrasound is highly sensitive for detecting clinically important vascular injuries, but it cannot evaluate the aerodigestive structures of the neck.

Test-taking tip: The stem of the question points to an aerodigestive injury, which makes it easy to eliminate A and D as answers because these diagnostic modalities are used to evaluate vascular structures.

53. Answer:D

BCVI can result from cervical hyperextension and rotation or hyperextension during rapid deceleration, which can result in intimal dissections, thromboses, pseudoaneurysm, fistulas, and transections.

Catheter angiography is the gold standard for diagnosis of vertebral artery injury in blunt cervical trauma; however, it is invasive, time-consuming, and not readily available in most EDs.

While catheter angiography remains the gold standard for diagnosis of vertebral artery injury in blunt cervical trauma, CTA is readily available in most EDs and has excellent specificity (97%), and there is growing literature supporting use of CTA in the diagnosis of vertebral artery injury in blunt cervical trauma. In patients with injury identified by CTA, no confirmatory angiographic evaluation is indicated.

MRI has been gaining acceptance for evaluation of BCVI; however, sensitivity and specificity are lower than CTA, and MRI is not recommended as the sole screening tool for BCVI.

Duplex ultrasonography is noninvasive and inexpensive and has no contrast exposure; however, it is highly operator dependent, has limited views of zones I and III, and may miss small lesions.

Test-taking tip: “Gold standard” is the key phrase in this question, suggesting an order modality that might not be the most commonly used diagnostic study.

54. Answer:A

Vascular injuries are the leading cause of death in patients with penetrating trauma. Vascular injuries are present in up to 40% of patients with penetrating neck trauma with arterial injuries (predominately the carotid artery) accounting for 45% of penetrating neck vascular injuries. Venous injuries are found in up to 20%of patients with penetrating neck trauma.

Zone II (cricoid cartilage to the angle of the mandible) is the most commonly injured area (not zone I) and is easily accessed surgically.

Exposure and vascular control of injuries in zone I and zone III is more difficult, and most patients will undergo angiography and endoscopy to determine the need for operative intervention. Traditionally, zone II injuries undergo surgical exploration.

Wounds that are superficial to the platysma do not risk damage to vital structures of the neck; therefore, no imaging is required, and observation is appropriate.

Test-taking tip: The question presents a patient who is unstable and hemorrhage, which points to A as the correct answer.

55. Answer:D

Your patient has compartment syndrome from intraarterial injection of heroin.

The classic symptoms of compartment syndrome are pain/pain with passive stretching, paresthesias, and pallor, followed by paralysis and pulselessness later in the course.

Hand compartment syndrome, however, may not be associated with paresthesias, and assessing response to passive movement may make the diagnosis more difficult. Compartment pressure measurement in the small compartments of the hand may also be difficult, and diagnosis is typically made on a clinical basis.

Normal compartment pressure is <10 mm Hg. The delta pressure (the diastolic blood pressure minus the intracompartmental pressure) most commonly used to diagnose acute compartment syndrome is 30 mm Hg.

Clinical exam findings may be sufficient in the correct clinical setting to diagnose compartment syndrome. You do not have to have all classic findings (pain/pain on passive stretching, paresthesias, pallor, paralysis, pulselessness) to diagnose compartment syndrome, and in fact, paralysis and pulselessness are late findings of compartment syndrome. Timely diagnosis and fasciotomy is essential for limb salvage.

Compartment syndrome is not a diagnosis of exclusion. Laboratory testing and imaging are of no benefit in the diagnosis of compartment syndrome and may delay diagnosis, which may lead to permanent neuropathy and muscle necrosis.

Test-taking tip: Compartment syndrome is a limb-threatening condition, so it is easy to eliminate C as an answer because it states that this process is a diagnosis of exclusion (lots of tests and time delay).

56. Answer:A

Rapid and appropriate initial management of an amputated part is vital to salvage and preservation of function. Preservation of amputated parts is indicated whenever there is potential for replantation.

Appropriate care of the stump is as follows:

1.

Remove jewelry and/or constrictive clothing

2.

Irrigate the wound with normal saline

3.

Wrap the wound with a pressure dressing

4.

Apply a splint to prevent further injury and elevate

Appropriate care of the amputated part is as follows:

2.

Rinse the amputated part with saline solution

3.

Wrap the amputated part in moist sterile gauze and place in a plastic bag or container. Do not immerse the amputated part in saline or hypotonic fluids. This may cause severe maceration and may make replantation technically more difficult.

4.

Place the amputated part in a container and cool with separate containers of ice. Do not place the amputated part directly in ice.

Debridement and dissection should only be performed by the reimplantation team.

You should not clamp arterial bleeders. This may make reimplantation more difficult.

Test-taking tip: The procedures described in B and D are not commonly done in the ED for amputations so can easily be eliminated as potential correct answers.

57. Answer:C

High-pressure injection wounds (typically 2000–10,000 psi) are orthopedic emergencies. Despite often normal postinjection exams, areas may become edematous, pale, and tender and require surgical decompression and debridement.

Obtain an emergent orthopedics consultation, immobilize and elevate the affected extremity, update tetanus, and administer antibiotics.

While it is appropriate to update tetanus, this high-pressure injection wound may become ischemic, requiring surgical intervention, and discharge home is not appropriate.

Radiographs can provide valuable information regarding type of injection (such as radiopaque substance) and subcutaneous air. Immobilization is indicated. High-pressure injection injuries are orthopedic emergencies, and it is not appropriate to delay orthopedic consultation.

Again, high-pressure injection injuries are orthopedic emergencies, and definitive treatment includes surgical decompression and debridement. Admitting for observation alone is not enough.

Test-taking tip: Three answers (A, B, D) are similar to each other, suggestion a nonemergent condition. Only C stands out as a different answer, making it most likely the correct answer.

58. Answer:C

Chest compartment syndrome is seen with full-thickness burns of the anterior and lateral chest wall or circumferential full-thickness burns of the chest wall. This can lead to severe restriction of chest wall motion and increased peak inspiratory pressures and difficulty ventilating. An escharotomy is essential in relieving chest wall restriction. Because full-thickness burns are insensate and involve coagulation of vessels, no anesthesia is needed. A properly executed escharotomy releases the eschar to the depth of the subcutaneous fat only and results in minimal bleeding. Perform an escharotomy of the chest by making incisions down the anterior axillary lines with cross-incisions at the junction of the thorax and abdomen.

The American Burn Association defines two requirements for the diagnosis of smoke inhalation injury: exposure to a combustible agent, and signs of exposure to smoke in the lower airway, below the vocal cords, by bronchoscopy. Treatment is supportive, and decreasing Fio2 in a patient with hypoxia and smoke inhalation injury would not be recommended.

While cyanide poisoning should be considered in all inhalation injuries, hydroxocobalamin, not methylene blue, is the antidote for cyanide poisoning.

Carbon monoxide (CO)poisoning should also be considered in all inhalation injuries, particularly in enclosed areas; however, this clinical scenario is most consistent with chest compartment syndrome with circumferential full thickness chest wall burns and increased peak inspiratory pressure.

Test-taking tip: The combination of a full-thickness burn to the chest and an increased peak inspiratory pressure should point to you the correct answer.

59. Answer:B

The Parkland formula [4 mL × body weight (kg) × burn surface area (BSA) (kg)] is used to determine the amount of crystalloid for infusion in the first 24 hours in patients who present with partial-thickness and full-thickness burns. Half of the volume is administered in the first 8 hours, and the remaining half is administered over the next 16 hours.

The extent of burns can be determined by using the Rule of Nines, in which the head represents 9% and anatomic regions represent 9% or multiples of 9% of the total BSA.

In this case, each leg represents 18% (9% anterior and 9% posterior), for a total of 36% BSA.

4 mL × body weight (kg) × BSA (%)

4 mL × 90 kg × 36% = 12,960 mL

One-half of 12,960 mL is 6,480 mL therefore, 6,480 mL over the first 8 hours with the remaining 6,480 mL over the next 16 hours.

Test-taking tip: Brush up on some basic formulas, such as Parkland, that are used clinically before taking the test.

60. Answer:B

Burn depth has traditionally been described in degrees: first, second, third, and fourth degrees; however, classification based on need for surgical intervention has become the accepted nomenclature.

Superficial burns involve only the epidermal layer. Skin is red, painful, and tender, with no blistering.

Partial-thickness burns extend into the dermis and are further divided into:

A.

Superficial partial-thickness: Involve the epidermis and superficial dermis and spare the deeper layers. Skin is blistered, and exposed dermis is red, moist, and painful. Scarring is minimal.

B.

Deep partial-thickness: Involve the deep dermal layer. Skin is blistered. Exposed dermis is pale and white to yellow, and burned areas do not blanch. Absent capillary refill and pain sensation. May be difficult to distinguish from full-thickness burns. Scarring is common. Debridement and skin grafting may be needed.

Full-thickness burns involve the entire thickness of the skin, and all epidermal and dermal layers are destroyed. Skin is charred, pale, painless, leathery. Surgical repair and skin grafting is necessary.

Test-taking tip: This is a “you know it or don’t” kind of question. If you don’t know it, choose an answer and move on. You can perhaps mark it for review when you are done at the end of the test. Don’t, however, let these kinds of questions get you stuck and waste your time.

61. Answer:A

Hydrofluoric acid is a corrosive inorganic acid often used in glass etching, electronic industries, and cleaning solutions. As fluoride ions complex with calcium and magnesium and in small, localized injuries, the main result is severe pain; however, large burns, inhalation injuries, or concentrated hydrofluoric acid exposure can lead to hypocalcemia, hypomagnesemia, hyperkalemia, cardiac arrhythmias, and death.

For minor exposures, like the one shown in the image, initial treatment consists of calcium gluconate gel to burned areas.

For poorly controlled pain, consider intradermal calcium, intraarterial calcium, or IV calcium (Bier block); for inhalation injuries, consider nebulized calcium; and for systemic toxicity (i.e., electrolyte abnormalities) and cardiac arrhythmias (such as prolonged QTc), treat with IV calcium.

This is not a thermal burn and does not require skin grafting.

Decontamination and copious irrigation with water (not normal saline) are recommended treatment for the majority of chemical burns with the exception of dry lime, elemental metals, and phenols. Water is not recommended for irrigation of dry lime, elemental metals, or phenol.

Phenol (carbolic acid) is moderately soluble in water, and irrigating with water can spread the chemical, causing increased absorption and toxicity. Polyethylene glycol (PEG) is used to remove phenol.

Test-taking tip: This is a “you know it or don’t” kind of question. If you don’t know it, choose an answer and move on. You can perhaps mark it for review when you are done at the end of the test. Don’t, however, let these kinds of questions get you stuck and waste your time.

62. Answer:C

Chemical burns cause progressive tissue damage until the chemical is inactivated or removed. Acids damage tissue by coagulation necrosis. This process limits the depth of penetration of tissues.

Alkalis react with lipids and damage tissue by liquefaction necrosis. This process permits penetration of the chemical deep into the tissues until it is neutralized.

Exposure to alkali is more likely to produce deep disuse injury.

Wet cement is an alkaline substance that contains mainly calcium oxide followed by oxides of silicon, aluminum, magnesium, sulfur, iron, and potassium. When combined with water, an exothermic reaction converts calcium oxide to calcium hydroxide, a strong corrosive alkali.

Acetic acid (vinegar), hydrochloric acid, and sulfuric acid are acids that damage tissue by coagulation necrosis and have limited depth of penetration.

Test-taking tip: Wet cement is the only answer that stands out as different (all the others are acids) making it most likely to be the correct answer.

63. Answer:D

Urinary retention.

Cauda equine syndrome is an epidural compression syndrome that may be caused by intervertebral disk herniation, tumor, spondylosis, or inflammatory conditions and may present with low back pain (with or without radiation), weakness, sensory loss, decreased lower extremity reflexes, and bowel and/or bladder dysfunction.

Of all of these findings, the most common finding is urinary retention with or without overflow incontinence with a sensitivity of 90% and specificity of about 95%.

Treatment consists of emergent MRI and neurosurgical consultation.

Test-taking tip: This is a “you know it or don’t” kind of question. If you don’t know it, choose an answer and move on. You can perhaps mark it for review when you are done at the end of the test. Don’t, however, let these kinds of questions get you stuck and waste your time.

64. Answer:B

S. aureus is the most common causative organism in all cases of osteomyelitis, including case of traumatic osteomyelitis after an open fracture.

Puncture wounds through tennis shoes have been implicated in pseudomonal osteomyelitis of the foot, but even in these cases S. aureus is more common.

Similarly, S. paratyphi is a consideration as a causative agent in patients with sickle cell disease, but S. aureus is still a more common cause in this patient population.

H. influenzae can cause osteomyelitis in children but is still less common than S. aureus, especially in vaccinated children.

Test-taking tip:H. influenzae and Salmonella infections are relatively rare and easy to eliminate, leaving two answers to choose from.

65. Answer:C

A Galeazzi fracture is a fracture of the distal third of the radius, usually shortened and displaced, with disruption of the distal radioulnar joint. It is more common in adults than children. The mechanism of injury is usually direct wrist trauma from a fall on outstretched hand (FOOSH), with the forearm in pronation. Clinically, there is often angulation of the radial forearm and mobility of the ulnar head.

A Colles fracture is a dorsally displaced radial metaphyseal fracture, also commonly caused by a FOOSH. It is the most common of all wrist fractures. On examination, the wrist will have the classic “dinner fork” or dorsiflexion deformity.

A fracture of the ulnar shift with associated radial head dislocation is referred to as a Monteggia fracture-dislocation. On examination, the forearm may appear angulated and shortened, and there are usually pain and swelling at the elbow.

A Smith fracture is also known as a reverse Colles and is a fracture of the distal radius with volar angulation of the distal fracture fragment. It can be caused by a FOOSH or direct blow to the dorsum of the wrist. The hand is palmarly displaced, creating a “garden spade” deformity.

The mnemonic MUGR (pronounced “mugger”) can help you remember the difference between a Monteggia and a Galeazzi fracture: Monteggia—Ulna fractured; and Galeazzi—Radius fractured.

Test-taking tip: Eponyms are commonly tested on standardized medical exams, so make time to review them as part of a study plan.

66. Answer:C

This is a Lisfranc injury, characterized by a fracture at the base of the second metatarsal or second cuneiform with dislocation of the lateral four metatarsals.

Direct trauma or hyperdorsiflexion of the joint between the midfoot and the forefoot (the Lisfranc joint) can result in this injury. It may result from a high-energy or low-energy mechanism and often results from an MVC, when the patient’s foot is plantar-flexed as it presses the brake.

Common physical exam findings include significant foot swelling, tenderness over the tarsometatarsal (TMT) joint, and plantar ecchymosis.

There is significant morbidity associated with delay in treatment for this injury; thus, Lisfranc injures require orthopedic consultation in the ED. This injury requires ORIF to improve outcomes and reduce complications.

Test-taking tip: You can usually eliminate answers that have the word “always” or “never” in them.

67. Answer:B

Bennett and Rolando fractures are two intraarticular fractures of the thumb base resulting from an axial force acting on a partially flexed metacarpal.

A Bennett fracture is a fracture-dislocation in which the volar lip of the metacarpal base continues to articulate with the trapezium, while the remainder of the metacarpal is dislocated at the carpometacarpal joint. (The small volar fragment is held in place by the volar oblique ligament.)

A Rolando fracture is a comminuted fracture of the base of the thumb metacarpal, and the fracture fragments typically form a “Y” shape.

A Rolando fracture has a worse prognosis than a Bennett fracture.

Both of these fractures require a thumb spica splint and immediate outpatient hand service referral because they will most likely require ORIF. Neither requires emergent hand service consultation in the ED.

Test-taking tip: Eponyms are commonly tested on standardized medical exams, especially in orthopedics, so make sure to review them as part of a study plan.

68. Answer:D

Flexor tenosynovitis is commonly the result of a puncture wound on the volar surface of a digit, although this is not one of Kanavel’s signs.

Kanavel’s signs include diffuse fusiform swelling (tapering at both ends because of the course of the tendon sheath), finger held in slightly flexion, severe pain on passive extension, and tenderness along the course of the tendon sheath.

Treatment is IV antibiotics, update of tetanus as indicated, and consultation of the hand service for surgical incision and drainage.

Test-taking tip: Brush up on key signs and symptoms patterns (e.g., Kanavel’s signs, Beck’s triad) before the test.

69. Answer:A

SCFE is a condition that occurs in adolescent children around the age of 10 to 16 years. This condition is characterized by displacement of the capital femoral epiphysis from the femoral neck. It is more common in boys than in girls and often occurs in obese children.

The clinical presentation is slowly progressive and is usually atraumatic, or it may be associated with remote trivial trauma. The child will typically present with a painless limp or may have pain in the knee, thigh, groin, or hip. The hip will be adducted and externally rotated, and on exam, there will be limitation of internal rotation, abduction, and flexion.

Treatment is operative to stabilize the physis and avoid the potential complications of avascular necrosis of the femoral head and chondrolysis (narrowing of the joint space).

Test-taking tip: Brush up on common syndromes that commonly present to the ED.

70. Answer:D

This radiograph shows a perilunate dislocation. The radiolunate articulation is preserved, but the lunate is displaced and rotated volarly, with the other carpal bones located dorsally.

In a lunate dislocation, the lunate loses its articulation with the radius and is displaced and rotated volarly. The remainder of the carpal bones are displaced dorsally, and the articulation of the radius, capitate, and third metacarpal is preserved. Both a perilunate and a lunate dislocation result from forced hyperextension, such as a fall on an outstretched hand (FOOSH).

The most commonly injured ligament of the wrist is the scapholunate ligament, leading to a scapholunate dislocation. This is diagnosed when there is >3 mm of widening of the scapholunate space on radiograph.

More forceful trauma can lead to further ligament disruption, resulting in a lunate or perilunate dislocation, which are commonly associated with fractures of the scaphoid or lunate. These injuries require emergent orthopedic hand consultation for possible ORIF.

De Quervain’s refers to a noninfectious overuse syndrome resulting in pain on the radial side of the wrist with pinching or wrist movements.

Test-taking tip: Review the anatomy of the hand, especially the carpal bones and their fracture-dislocation patterns, because they are frequently tested.

71. Answer:C

This radiograph shows a posterior shoulder dislocation. Posterior dislocations are uncommon, accounting for fewer than 5% of all glenohumeral dislocations. A posterior dislocation can occur after a FOOSH with the arm held in flexion, adduction, and internal rotation or after a direct blow to the anterior aspect of the shoulder. Convulsive seizures due to either epilepsy or electric shock are associate with this type of dislocation.

A posterior dislocation is a commonly missed diagnosis because the radiographic findings can be subtle. Clinically, the patient will typically present with the arm held in adduction and internal rotation. Radiographically, the anterior-posterior view will show loss of the normal elliptical overlap of the humeral head and the posterior glenoid rim, as well as the humerus in internal rotation, the “lightbulb” or “rifle barrel” sign (as seen in this film). The axillary “Y” view is more diagnostic.

Posterior shoulder dislocations are less commonly associated with neurovascular injuries compared with anterior shoulder dislocations, which may result in injury to the axillary nerve and artery. Posterior shoulder dislocations, however, are more commonly associated with fractures, including fracture of the lesser tuberosity or neck of the humerus, fracture of the posterior aspect of the glenoid rim, and impaction fracture of the anterior aspect of the humeral head (reverse Hill-Sachs deformity), as well as injuries to the labrum and the rotator cuff.

Test-taking tip: This is a “you know it or don’t” kind of question. If you don’t know it, choose an answer and move on. You can perhaps mark it for review when you are done at the end of the test. Don’t, however, let these kinds of questions get you stuck and waste your time.

72. Answer:B

A boutonnière deformity may result if the extensor mechanism overlying the PIP is damaged by a hyperflexion injury. The central extensor tendon attaches to the middle phalanx; thus, injury will disrupt extension of the PIP, but because the lateral bands of the extensor mechanism insert on the distal phalanx and remain intact, this will result in flexion at the PIP and extension of the DIP. There may be an associated dorsal chip avulsion fracture of the base of the middle phalanx. Treatment is to splint the PIP in extension for 4 to 6 weeks.

A mallet finger may also result from a jammed finger and refers to disruption of the terminal portion of the extensor tendon, which inserts into the distal phalanx and disrupts extension of the DIP. This may be associated with avulsion of the dorsal base of the distal phalanx. Treatment is to splint the DIP in extension for 6 weeks.

A gamekeeper’s, or skier’s, thumb results from forced abduction of the metacarpophalangeal (MCP) joint of the thumb and causes rupture of the ulnar collateral ligament. The patient will have tenderness along the ulnar aspect of the thumb and laxity on valgus stress of the MCP. Treatment of a partial tear involves a thumb spica splint, and treatment of a complete tear requires surgery to prevent chronic instability.

The volar plate is a thick ligament between the proximal phalanx and middle phalanx. This can be ruptured in a hyperextension injury of the finger. The patient will have swelling and tenderness over the volar aspect of the PIP and may have an associated avulsion fracture of the volar aspect of the middle phalanx. This is treated with splinting the finger in slight flexion.

Test-taking tip: This is a “you know it or don’t” kind of question. If you don’t know it, choose an answer and move on. You can perhaps mark it for review when you are done at the end of the test. Don’t, however, let these kinds of questions get you stuck and waste your time.

73. Answer:C

This patient has physical exam findings suggestive of rupture to the anterior and posterior cruciate ligament. This should raise the clinical suspicion for a spontaneously reduced knee dislocation, especially given the high-velocity mechanism.

Popliteal artery injury is common in posterior knee dislocations and potentially limb-threatening. The exam must include evaluation of the popliteal pulse, pedal pulses, and ankle/brachial index (ABI). If there are obvious vascular deficits, the patient should go straight to the operating room, and if equivocal, a CTA should be obtained. There may also be injury to the peroneal nerve and, less commonly, the tibial nerve. Damage to the peroneal nerve will result in weakness of foot and ankle dorsiflexion (i.e., footdrop and weakness of toe extension). Damage to the tibial nerve will result in weakness of inversion and plantar flexion.

Although MRI may help to further characterize internal derangement to an injured knee that is not fractured, it is not usually emergently indicated in the ED.

CT without contrast may help to pick up occult fractures, but this is not the concern in this scenario.

Radiographs taken above and below a fractured joint may help to pick up missed injuries, but this patient does not have a fracture, so this is probably not indicated.

Test-taking tip: The key word in this question is “acute” management, suggesting a test that will discover an injury that may have significant morbidity. CTA is the only answer that fits this setting.

74. Answer:B

The earliest symptom in compartment syndrome is pain out of proportion to the injury associated with tense compartments. This is followed by the “5 Ps”—pain, pallor, paresthesias, paralysis, and pulselessness.

Compartment syndrome can result from trauma (fracture, crush injury, high-pressure injection injury, prolonged compression, hematoma) but may also result from other mechanisms, including infection.

The injury most commonly associated with compartment syndrome is a proximal tibial shaft fracture, and the anterior tibial compartment is most commonly involved. There are four compartments in the leg: anterior compartment, lateral compartment, superficial posterior, and deep posterior compartment. The anterior compartment contains the deep peroneal nerve, the anterior tibial artery and vein, the tibialis anterior muscle, and the extensor muscles of the foot. Associated findings of compression of the anterior compartment would be loss of sensation between the great toe and second toe and weakness of dorsiflexion of the great toe and foot.

A normal compartment pressure is 0 to 8 mm Hg. Compartment pressure >30 mm Hg is indication of injury, while a pressure >40 mm Hg is indication for fasciotomy.

Test-taking tip: Brush up on the clinical findings of common syndromes that present to the ED.

75. Answer:D

This is a Jones fracture, which is a transverse fracture of the fifth metatarsal at the level of the intermetatarsal joint. It is common in athletes playing running/jumping sports, when there is a transverse force applied to a plantar-flexed foot.

These fractures have a high incidence of nonunion and thus are managed either operatively or, if nondisplaced, conservatively in a non–weight-bearing cast for a minimum of 6 weeks.

In contrast, a pseudo-Jones or “dancer’s” fracture is an avulsion of the base of the fifth metatarsal by the peroneus brevis tendon. This usually results from a plantar flexion/inversion injury, as is common in an ankle sprain. Thus, tenderness at the base of the fifth metatarsal is part of the Ottawa ankle rules. This fracture heals well and needs only a rigid soled shoe (cast shoe).

Test-taking tip: Eponyms are commonly tested on standardized medical exams, especially in orthopedics, so make sure to review them as part of a study plan.

76. Answer:D

This radiograph shows displacement of the lateral mortise due to rupture of the deltoid ligament. Any time a medial malleolus fracture or injury to the deltoid ligament is suspected, a Maisonneuve fracture (fracture of the proximal third of the fibula associated with rupture of the deltoid ligament or fracture of the medial malleolus and disruption of the syndesmosis) must be ruled out. The associated proximal fibula fracture is shown in Figure 18.30. Any time a patient has medial ankle tenderness and swelling, an associated proximal fibula fracture must be ruled out.

Figure 18.30

A client is admitted to the emergency department following a fall from a horse

Knee radiograph.

The mechanism is usually eversion, not inversion, of the dorsiflexed foot in a high-intensity sport. This mechanism can result in a “high ankle sprain,” which may be associated with rupture of the anterior tibiofibular ligament and injury of the syndesmosis. If there is a positive “squeeze test” or “dorsiflexion external rotation” test, this patient may need stress radiographs by orthopedics/podiatry to rule out ligamentous instability. If weight-bearing films are possible, this will increase the sensitivity for ligamentous instability by showing displacement of the medial aspect of the mortise (increase in the width of the medial clear space).

The lateral ligament complex is usually injured by an inversion stress to the ankle.

Test-taking tip: Answers B and D are contradictory to each other, increasing the likelihood that one of them is the correct answer.

77. Answer:A

A nondisplaced spiral fracture of the distal tibial shaft is also called a “toddler’s fracture” and is very common in ambulatory children with low-mechanism trauma. This injury is not suspicious for nonaccidental trauma. The fracture line may be difficult to see and may be mistaken for a nutrient vessel. Bony callus formation at follow-up confirms the diagnosis.

A “bucket handle” fracture, shown in Figure 18.31, is also called a metaphyseal “corner” fracture and results from intentional pulling and twisting. The shearing forces result in separation of the subperiosteal bone collar of the metaphysis, resulting in the disk-shaped fragment described as a “bucket handle.” This is very concerning for nonaccidental trauma, especially in a nonambulatory child.

Figure 18.31

A client is admitted to the emergency department following a fall from a horse

Knee radiograph.

The most common fractures in nonaccidental trauma are transverse or spiral diaphyseal fractures of the long bones in a nonambulatory child. The physician should maintain a high suspicion of nonaccidental trauma in any fracture in a nonambulatory child, in a new presentation of an old fracture, in a child with multiple fractures at different stages, and when the clinical history does not seem to fit the expected mechanism of injury.

Test-taking tip: This is a “you know it or don’t” kind of question. If you don’t know it, choose an answer and move on. You can perhaps mark it for review when you are done at the end of the test. Don’t, however, let these kinds of questions get you stuck and waste your time.

78. Answer:D

Thoracic outlet syndrome is characterized by compression of the neurovascular bundle (brachial plexus, subclavian artery, and/or subclavian vein) as it courses through the thoracic outlet. Symptoms include pain, paresthesias/numbness (neurologic), swelling (venous), and arm ischemia (arterial). The syndrome may be associated with a cervical rib, muscular anomalies, or injury.

The most reliable test is the EAST. This is performed by having the patient raise the hands above the head and open and close the fist for 3 minutes. Inability to complete the test due to pain or paresthesias is a positive test. Adson’s test evaluates specifically for arterial thoracic outlet syndrome (not neurologic or venous). In this test, both radial pulses are palpated while the patient turns the head from side to side. Loss of the radial pulse is a positive test. If conservative therapy (physical therapy, weight loss) fails, thoracic outlet decompression may be needed.

Adhesive capsulitis or “frozen shoulder” is characterized by limited range of motion of the affected shoulder and trouble with activities of daily living. On passive testing of external rotation, a sense of mechanical restriction of joint motion can often be appreciated.

Acute rotator cuff tears present with point tenderness over the site of the tear (the greater tuberosity). The drop-arm test, performed by passively abducting the arm to 90 degrees and asking the patient to hold the arm in this position, is positive with significant tears. Slight pressure on the distal forearm or wrist causes the patient to drop the arm suddenly.

Impingement syndrome of the shoulder is a spectrum of illness and is marked by progression of symptoms. The Hawkins-Kennedy impingement sign involves placing the arm into 90 degrees of flexion followed by internal rotation; the test is consider positive if the patient has pain with internal rotation.

Test-taking tip: This is a “you know it or don’t” kind of question. If you don’t know it, choose an answer and move on. You can perhaps mark it for review when you are done at the end of the test. Don’t, however, let these kinds of questions get you stuck and waste your time.

79. Answer:B

This radiograph shows a Salter-Harris II fracture, which extends through the metaphysis. The Salter-Harris classification refers to physial, or growth plate, fractures. Salter-Harris II is the most common, representing more than 75% of growth plate fractures. The system classifies fractures from I to V, generally with I being the least complex and V being the most complex. Complications can include premature physial fusion and growth disturbance. A mnemonic to remember the classification is as follows:

S – slipped—across physis, or nothing seen radiographically = Salter-Harris I

A – above the growth plate—through growth plate and metaphysis = Salter-Harris II

L – lower than the growth plate—through growth plate and epiphysis = Salter-Harris III

T – through the growth plate—through epiphysis, growth plate, and metaphysis = Salter-Harris IV

R – rammed—growth plate is crushed = Salter-Harris V

Test-taking tip: Brush up on your Salter-Harris classifications because they are frequently tested on emergency medicine standardized tests.

80. Answer:D

This radiograph shows a large elbow effusion that presents with a posterior fat pad, which is always pathologic for an occult fracture, as well as an anterior sail sign. A normal elbow radiograph may have a small anterior fat pad, but a large anterior sail sign, as shown in this radiograph, indicates an elbow effusion. This can be the only radiographic abnormality seen with a radial head fracture, usually in adults, or a supracondylar fracture in children.

Test-taking tip: This is a “you know it or don’t” kind of question. If you don’t know it, choose an answer and move on. You can perhaps mark it for review when you are done at the end of the test. Don’t, however, let these kinds of questions get you stuck and waste your time.

81. Answer:C

Finkelstein’s test refers to ulnar deviation of the fisted hand and will reproduce the pain in de Quervain’s tenosynovitis. De Quervain’s tenosynovitis is an overuse inflammatory condition that causes pain on the radial side of wrist. This is due to inflammation of the abductor pollicis longus and the extensor pollicis brevis as they pass through the first dorsal compartment of the wrist. Treatment is with thumb spica splint, NSAIDs, or in severe cases, surgery.

Tinel’s test, percussion of the median nerve at the wrist, and Phalen’s test, maximal palmar flexion at the wrist, both refer to tests for carpal tunnel syndrome that have recently come under some scrutiny for not having adequate sensitivity or specificity.

Thompson’s test is used to detect Achilles tendon rupture. If the foot does not plantar-flex when the calf is squeezed, this is a positive Thompson’s test and is consistent with an Achilles tendon rupture.

Test-taking tip: Brush up on your musculoskeletal tests and signs because they are frequently tested on emergency medicine standardized tests.

82. Answer:D

When a child presents with pain and/or a limp, it is important to consider all of the diagnoses listed. Toxic (transient) synovitis is the most common cause of a painful hip in children, while septic arthritis is the most common cause of a painful hip in infants.

The patient with toxic synovitis is usually 3 to 8 years of age and presents with an acutely painful hip or knee and a limp. These patients are generally well-appearing but may have a slightly elevated temperature and erythrocyte sedimentation rate (ESR). They respond well to NSAIDs. This is, however, a diagnosis of exclusion, after a septic joint has been ruled out.

The patient with septic arthritis will typically present acutely ill-appearing, have a fever, and have an elevated ESR and white blood cell count. The most common organism overall is S. aureus. Salmonella is characteristic in patients with sickle cell disease. Neisseria gonorrhoeae should be suspected in adolescents.

The characteristic patient with SCFE is an obese adolescent male who presents with insidious onset of hip, groin, thigh, or knee pain.

The patient with Legg-Calvé-Perthes disease is typically 4 to 8 years old and presents with insidious onset of pain and limp due to idiopathic avascular necrosis of the femoral head. Temperature and ESR will be normal. If radiographs are normal and the patient cannot bear weight, consider obtaining an MRI because plain films may be normal early in the course of the disease.

Test-taking tip: This is a “you know it or don’t” kind of question. If you don’t know it, choose an answer and move on. You can perhaps mark it for review when you are done at the end of the test. Don’t, however, let these kinds of questions get you stuck and waste your time.

83. Answer:D

This radiograph shows a Lisfranc injury, which is characterized by disruption of the TMT joint. The Lisfranc ligament connects the medial cuneiform to the second metatarsal and provides midfoot stability. This may or may not be associated with a fracture. In this case, it is not, and the radiographic abnormality is the increased space between the first and second metatarsals. Any time there is a fracture of the base of the second metatarsal (where the Lisfranc ligament attaches), a Lisfranc injury should be suspected. If the patient can tolerate a weight-bearing radiograph, this will help in making the diagnosis by making any space between the first and second metatarsals more evident. This injury needs urgent podiatry or orthopedic consultation to determine whether it will be managed conservatively (6 weeks in a non–weight-bearing cast) or operatively.

Test-taking tip: When one answer (urgent consultation) stands out for three similar answers (all weight-bearing immobilization methods), the different answer is probably correct.

84. Answer:C

This patient likely has a significant crush injury, based on mechanism, which is a relative contraindication to replantation because the tissues and neurovasculature are likely too severely damaged to undergo successful repair. That said, all digit amputations should be treated with rapid consultation with a hand surgeon. All patients are candidates for surgical repair until the surgeon deems otherwise. Tetanus and antibiotic prophylaxis should be given. The amputated part should be wrapped in moist gauze and placed in a clean, sealed plastic bag or specimen cup; then the sealed bag or cup should be placed in ice.

Some of the indications for replantation include amputation of the thumb at any level, amputation of multiple digits, virtually all pediatric amputations, and hand amputations through the palm and distal wrist. Some relative contraindications include severely crushed parts, amputations at multiple levels, and prolonged time of ischemia.

Test-taking tip: When one answers stands out as different from the others (run over by a tractor vs. all other minor mechanisms), it is probably the correct answer.

85. Answer:B

This is a Monteggia fracture, which is a fracture of the proximal third of the ulnar shaft associated with radial head dislocation. A Monteggia fracture is more common in children than in adults. It can be associated with a radial nerve injury. In pediatrics, there may be a bowing/plastic deformity of the ulna rather than a fracture associated with a radial head dislocation, so it is important to pay attention to the radiocapitellar line. It is very uncommon to see an isolated radial head dislocation in children (though it is common to see a subluxation of the radial head, i.e., nursemaid’s elbow). Treatment in adults is ORIF; however, in children, it is usually closed reduction and immobilization.

A Galeazzi fracture is a fracture of the shaft of the radius with dislocation of the distal radioulnar joint. The ligaments of the inferior radioulnar joint are ruptured, and the head of the ulna is displaced from the ulnar notch of the radius.

A nightstick fracture is a fracture of the ulna, radius, or both. It is usually a defensive injury when a person uses the forearm to self-protect from a blow or a solid object.

Test-taking tip: You can eliminate choice A because its description of the fracture is not consistent with the radiograph. Study up on your fracture eponyms because they are commonly tested.

86. Answer:A

The Ottawa ankle rules include all of the above except A. The rules include tenderness along the posterior edge of the distal 6 cm of the medial or lateral malleolus, not the anterior edge. These rules have been validated by numerous clinical studies and provide guidelines for which patients do not need a radiograph of the ankle if they have none of the criteria.

Test-taking tip: Brush up on common rules such as Ottawa and NEXUS because they are commonly tested on emergency medicine standardized tests.

87. Answer:B

This radiograph shows a lateral tibial plateau fracture. The degree of depression and whether there is any associated ligamentous injury will help determine whether surgical management is indicated.

A tibial plateau fracture and any tibia or fibula fracture may be associated with compartment syndrome

Lateral tibial plateau fractures may be associated with a deep peroneal nerve injury; thus, the physician should examine and document sensation in the first dorsal webspace of the foot (between the great toe and second toe) and evaluate the patient’s ability to dorsiflex the foot (rule out footdrop).

The most common complication is post-traumatic arthritis.

Test-taking tip: This is a “you know it or don’t” kind of question. If you don’t know it, choose an answer and move on. You can perhaps mark it for review when you are done at the end of the test. Don’t, however, let these kinds of questions get you stuck and waste your time.

88. Answer:A

A nightstick fracture is a nondisplaced fracture of the ulnar shaft (it can also involve the radial shaft). A concomitant radial head dislocation (Monteggia fracture-dislocation) must be ruled out. A nightstick fracture may be associated with a radial nerve injury, and complications include nonunion.

A Colles fractures is of the distal radius with dorsal displacement and volar angulation, with or without an ulnar styloid fracture.

A Galeazzi fracture is a fracture of the shaft of the radius with dislocation of the distal radioulnar joint. The ligaments of the inferior radioulnar joint are ruptured, and the head of the ulna is displaced from the ulnar notch of the radius.

The Salter-Harris classification (I–V) refers to physial, or growth plate, fractures in children.

Test-taking tip: Brush up on your fracture eponyms because they are commonly tested on emergency medicine standardized tests.

89. Answer:C

With tenderness in the anatomic snuff box, one must assume that this patient has a scaphoid fracture. The scaphoid is the most commonly fractured carpal bone. Initial radiographs can be negative, so any patient with tenderness in the anatomic snuff box should be placed in a thumb spica splint and have repeat radiographs in 2 weeks. If at 2 weeks a fracture is still clinically suspected but the radiograph remains negative, MRI should be ordered.

This is an important injury not to miss because there is a high incidence of avascular necrosis owing to the poor vascularity of the scaphoid. This is especially true of injuries to the proximal portion of the scaphoid, which is the least well vascularized, because blood supply flows from the distal to the proximal portion of the bone.

Test-taking tip: This is a “you know it or don’t” kind of question. If you don’t know it, choose an answer and move on. You can perhaps mark it for review when you are done at the end of the test. Don’t, however, let these kinds of questions get you stuck and waste your time.

90. Answer:B

A forearm fracture of both bones requires a significant amount of force; thus, these fractures are usually displaced. This usually requires ORIF. The most significant complication is compartment syndrome. Other complications include reduced ability to pronate and supinate, nonunion, and neurovascular injury.

Supracondylar fractures are most common in children between the ages of 5 and 10 years and area associated with nerve injuries, with the anterior interosseous nerve being the most commonly injured.

Monteggia fracture-dislocation is a fracture of the ulna diaphysis with anterior dislocation of the radial head. Complications include malunion, nonunion, synostosis, stiffness, and nerve palsy.

Galeazzi fracture-dislocation involves a fracture of the distal third of the radius with an associated dislocation of the distal radioulnar joint. Complications include malunion, nonunion, limited range of motion, and chronic pain.

Test-taking tip: Brush up on common causes of common presentations, such as compartment syndrome.

91. Answer:B

Thompson’s test is performed with the patient prone and the knee flexed to 90 degrees. The calf is then squeezed, and the foot should passively plantar-flex. If there is no passive plantar flexion, this is a positive Thompson’s test and indicates a complete tear of the Achilles tendon.

Thompson’s test should be negative (the foot does passively plantar-flex with calf squeeze) with a gastrocnemius rupture.

Both an Achilles tendon rupture and gastrocnemius rupture occur with sudden contraction of the calf.

With a patellar tendon rupture, the patient will be unable to extend the leg or maintain a passively extended leg in extension. A patellar tendon rupture occurs with sudden contraction of the quadriceps, as occurs in jumping sports.

Test-taking tip: Brush up on your musculoskeletal tests and signs because they are frequently tested on emergency medicine standardized tests.

92. Answer:B

This is a torus fracture, which is an incomplete fracture characterized by buckling of the cortex. Children’s bones are soft and pliable and tend to bend before they break.

A greenstick fracture is a fracture of a long bone in children where there is disruption of only one side of the cortex. Torus fractures are much more common than greenstick fractures.

A plastic deformity is when a bone bends but does not break. This usually occurs in the radius and ulna.

A Salter-Harris fracture is a fracture through the growth plate.

Test-taking tip: Torus fractures are subtle. Practice reading some pediatric radiographs before the test.

93. Answer:A

The most commonly injured nerve with a humeral shaft fracture is the radial nerve. The radial nerve supplies sensation for the first dorsal webspace of the hand and is responsible for thumb and wrist extension.

Choice B describes the median nerve function. Choice C describes the ulnar nerve function. Choice D is an incorrect answer and does not describe a particular peripheral nerve function.

Test-taking tip: Brush up on major nerve sensation and function.

94. Answer:B

This radiograph shows osteochondritis dissecans, an idiopathic process that occurs in adolescents and results in separation of a bone fragment from the articular cartilage. This bone fragment can result in a joint locking. Osteochondritis dissecans most commonly affects the medial condyle of the femur. It may also affect the elbow (at the capitellum) or the ankle (at the talus). Initial treatment is generally nonoperative and consists of immobilization and limited weight bearing.

Test-taking tip: This is a “you know it or don’t” kind of question. If you don’t know it, choose an answer and move on. You can perhaps mark it for review when you are done at the end of the test. Don’t, however, let these kinds of questions get you stuck and waste your time.